Difference between revisions of "1991 AHSME Problems"

Line 1: Line 1:
Link to full test below.
+
== Problem 1 ==
  
http://www.artofproblemsolving.com/Forum/resources.php?c=182&cid=44&year=1991&sid=23c56fcb006bee49b72650509fbedb9b
+
If for any three distinct numbers <math>a</math>, <math>b</math>, and <math>c</math> we define <math>f(a,b,c)=\frac{c+a}{c-b}</math>, then <math>f(1,-2,-3)</math> is
 +
 
 +
<math> \textbf {(A) } -2 \qquad \textbf {(B) } -\frac{2}{5} \qquad \textbf {(C) } -\frac{1}{4} \qquad \textbf {(D) } \frac{2}{5} \qquad \textbf {(E) } 2 </math>
 +
 
 +
 
 +
[[1991 AHSME Problems/Problem 1|Solution]]
 +
 
 +
== Problem 2 ==
 +
 
 +
<math>|3-\pi|=</math>
 +
 
 +
<math> \textbf{(A)\ }\frac{1}{7}\qquad\textbf{(B)\ }0.14\qquad\textbf{(C)\ }3-\pi\qquad\textbf{(D)\ }3+\pi\qquad\textbf{(E)\ }\pi-3 </math>
 +
 
 +
 
 +
[[1991 AHSME Problems/Problem 2|Solution]]
 +
 
 +
== Problem 3 ==
 +
 
 +
<math>(4^{-1}-3^{-1})^{-1}=</math>
 +
 
 +
(A) <math>-12</math>  (B) <math>-1</math>  (C) <math>\frac{1}{12}</math>  (D) <math>1</math>  (E) <math>12</math>
 +
 
 +
[[1991 AHSME Problems/Problem 3|Solution]]
 +
 
 +
== Problem 4 ==
 +
 
 +
Which of the following triangles cannot exist?
 +
 
 +
(A) An acute isosceles triangle (B) An isosceles right triangle (C) An obtuse right triangle (D) A scalene right triangle (E) A scalene obtuse tr
 +
iangle
 +
 
 +
 
 +
[[1991 AHSME Problems/Problem 4|Solution]]
 +
 
 +
== Problem 5 ==
 +
 
 +
<asy>
 +
draw((0,0)--(2,2)--(2,1)--(5,1)--(5,-1)--(2,-1)--(2,-2)--cycle,dot);
 +
MP("A",(0,0),W);MP("B",(2,2),N);MP("C",(2,1),S);MP("D",(5,1),NE);MP("E",(5,-1),SE);MP("F",(2,-1),NW);MP("G",(2,-2),S);
 +
MP("5",(2,1.5),E);MP("5",(2,-1.5),E);MP("20",(3.5,1),N);MP("20",(3.5,-1),S);MP("10",(5,0),E);
 +
</asy>
 +
 
 +
In the arrow-shaped polygon [see figure], the angles at vertices <math>A,C,D,E</math> and <math>F</math> are right angles, <math>BC=FG=5, CD=FE=20, DE=10</math>, and <math>AB=AG</math>. The area of the polygon is closest to
 +
<math>\text{(A) } 288\quad
 +
\text{(B) } 291\quad
 +
\text{(C) } 294\quad
 +
\text{(D) } 297\quad
 +
\text{(E) } 300</math>
 +
 
 +
[[1991 AHSME Problems/Problem 5|Solution]]
 +
 
 +
== Problem 6 ==
 +
 
 +
 
 +
If <math>x\geq 0</math>, then <math>\sqrt{x\sqrt{x\sqrt{x}}}=</math>
 +
 
 +
(A) <math>x\sqrt{x}</math> (B) <math>x \sqrt[4]{x}</math> (C) <math>\sqrt[8]{x}</math> (D) <math>\sqrt[8]{x^3}</math> (E) <math>\sqrt[8]{x^7}</math>
 +
 
 +
[[1991 AHSME Problems/Problem 6|Solution]]
 +
 
 +
== Problem 7 ==
 +
 
 +
If <math>x=\frac{a}{b}</math>, <math>a\neq b</math> and <math>b\neq 0</math>, then <math>\frac{a+b}{a-b}=</math>
 +
 
 +
(A) <math>\frac{x}{x+1}</math> (B) <math>\frac{x+1}{x-1}</math> (C) <math>1</math> (D) <math>x-\frac{1}{x}</math> (E) <math>x+\frac{1}{x}</math>
 +
 
 +
[[1991 AHSME Problems/Problem 7|Solution]]
 +
 
 +
== Problem 8 ==
 +
 
 +
Liquid <math>X</math> does not mix with water. Unless obstructed, it spreads out on the surface of water to form a circular film <math>0.1</math>cm thick. A rectangular box measuring <math>6</math>cm by <math>3</math>cm by <math>12</math>cm is filled with liquid <math>X</math>. Its contents are poured onto a large body of water. What will be the radius, in centimeters, of the resulting circular film?
 +
 
 +
(A) <math>\frac{\sqrt{216}}{\pi}</math> (B) <math>\sqrt{\frac{216}{\pi}}</math> (C) <math>\sqrt{\frac{2160}{\pi}}</math> (D) <math>\frac{216}{\pi}</math> (E) <math>\frac{2160}{\pi}</math>
 +
 
 +
 
 +
[[1991 AHSME Problems/Problem 8|Solution]]
 +
 
 +
== Problem 9 ==
 +
 
 +
From time <math>t=0</math> to time <math>t=1</math> a population increased by <math>i\%</math>, and from time <math>t=1</math> to time <math>t=2</math> the population increased by <math>j\%</math>. Therefore, from time <math>t=0</math> to time <math>t=2</math> the population increased by
 +
 
 +
<math>\text{(A) (i+j)\%} \quad
 +
\text{(B) } ij\%\quad
 +
\text{(C) } (i+ij)\%\quad
 +
\text{(D) } \left(i+j+\frac{ij}{100}\right)\%\quad
 +
\text{(E) } \left(i+j+\frac{i+j}{100}\right)\%</math>
 +
 
 +
[[1991 AHSME Problems/Problem 9|Solution]]
 +
 
 +
== Problem 10 ==
 +
 
 +
Point <math>P</math> is <math>9</math> units from the center of a circle of radius <math>15</math>. How many different chords of the circle contain <math>P</math> and have integer lengths?
 +
 
 +
(A) 11  (B) 12  (C) 13  (D) 14  (E) 29
 +
 
 +
[[1991 AHSME Problems/Problem 10|Solution]]
 +
 
 +
== Problem 11 ==
 +
Jack and Jill run 10 km. They start at the same point, run 5 km up a hill, and reurn to the starting point by the same route. Jack has a 10 minute head start and runs at the rate of 15 km/hr uphill and 20 km/hr downhill. Jill runs 16 km/hr uphill and 22 km/hr downhill. How far from the top of the hill are they when they pass each other going in opposite directions (in km)?
 +
 
 +
<math>\text{(A) } \frac{5}{4}\quad
 +
\text{(B) } \frac{35}{27}\quad
 +
\text{(C) } \frac{27}{20}\quad
 +
\text{(D) } \frac{7}{3}\quad
 +
\text{(E) } \frac{28}{49}</math>
 +
 
 +
[[1991 AHSME Problems/Problem 11|Solution]]
 +
 
 +
== Problem 12 ==
 +
 
 +
The measures (in degrees) of the interior angles of a convex hexagon form an arithmetic sequence of integers. Let <math>m</math> be the measure of the largest interior angle of the hexagon. The largest possible value of <math>m</math>, in degrees, is
 +
 
 +
(A) 165  (B) 167 (C) 170 (D) 175 (E) 179
 +
 
 +
[[1991 AHSME Problems/Problem 12|Solution]]
 +
 
 +
== Problem 13 ==
 +
 
 +
Horses <math>X,Y</math> and <math>Z</math> are entered in a three-horse race in which ties are not possible. The odds against <math>X</math> winning are <math>3:1</math> and the odds against <math>Y</math> winning are <math>2:3</math>, what are the odds against <math>Z</math> winning? (By "odds against <math>H</math> winning are <math>p:q</math>" we mean the probability of <math>H</math> winning the race is <math>\frac{q}{p+q}</math>.)
 +
 
 +
<math>\text{(A) } 3:20\quad
 +
\text{(B) } 5:6\quad
 +
\text{(C) } 8:5\quad
 +
\text{(D) } 17:3\quad
 +
\text{(E) } 20:3</math>
 +
 
 +
[[1991 AHSME Problems/Problem 13|Solution]]
 +
 
 +
== Problem 14 ==
 +
 
 +
 
 +
If <math>x</math> is the cube of a positive integer and <math>d</math> is the number of positive integers that are divisors of <math>x</math>, then <math>d</math> could be
 +
 
 +
(A) <math>200</math>  (B) <math>201</math>  (C) <math>202</math>  (D) <math>203</math>  (E) <math>204</math>
 +
 
 +
[[1991 AHSME Problems/Problem 14|Solution]]
 +
 
 +
== Problem 15 ==
 +
 
 +
A circular table has 60 chairs around it. There are <math>N</math> people seated at this table in such a way that the next person seated must sit next to someone. What is the smallest possible value for <math>N</math>?
 +
 
 +
<math>\text{(A) } 15\quad
 +
\text{(B) } 20\quad
 +
\text{(C) } 30\quad
 +
\text{(D) } 40\quad
 +
\text{(E) } 58</math>
 +
 
 +
[[1991 AHSME Problems/Problem 15|Solution]]
 +
 
 +
== Problem 16 ==
 +
 
 +
One hundred students at Century High School participated in the AHSME last year, and their mean score was 100. The number of non-seniors taking the AHSME was <math>50\%</math> more than the number of seniors, and the mean score of the seniors was <math>50\%</math> higher than that of the non-seniors. What was the mean score of the seniors?
 +
 
 +
(A) <math>100</math> (B) <math>112.5</math> (C) <math>120</math> (D) <math>125</math> (E) <math>150</math>
 +
 
 +
[[1991 AHSME Problems/Problem 16|Solution]]
 +
 
 +
== Problem 17 ==
 +
 
 +
A positive integer <math>N</math> is a ''palindrome'' if the integer obtained by reversing the sequence of digits of <math>N</math> is equal to <math>N</math>. The year 1991 is the only year in the current century with the following 2 properties:
 +
 
 +
(a) It is a palindrome
 +
(b) It factors as a product of a 2-digit prime palindrome and a 3-digit prime palindrome.  
 +
 
 +
How many years in the millenium between 1000 and 2000 have properties (a) and (b)?
 +
 
 +
<math>\text{(A) } 1\quad
 +
\text{(B) } 2\quad
 +
\text{(C) } 3\quad
 +
\text{(D) } 4\quad
 +
\text{(E) } 5</math>
 +
 
 +
[[1991 AHSME Problems/Problem 17|Solution]]
 +
 
 +
== Problem 18 ==
 +
 
 +
If <math>S</math> is the set of points <math>z</math> in the complex plane such that <math>(3+4i)z</math> is a real number, then <math>S</math> is a
 +
 
 +
(A) right triangle
 +
(B) circle
 +
(C) hyperbola
 +
(D) line
 +
(E) parabola
 +
 
 +
 
 +
[[1991 AHSME Problems/Problem 18|Solution]]
 +
 
 +
== Problem 19 ==
 +
 
 +
<asy>
 +
draw((0,0)--(0,3)--(4,0)--cycle,dot);
 +
draw((4,0)--(7,0)--(7,10)--cycle,dot);
 +
draw((0,3)--(7,10),dot);
 +
MP("C",(0,0),SW);MP("A",(0,3),NW);MP("B",(4,0),S);MP("E",(7,0),SE);MP("D",(7,10),NE);
 +
</asy>
 +
 
 +
Triangle <math>ABC</math> has a right angle at <math>C, AC=3</math> and <math>BC=4</math>. Triangle <math>ABD</math> has a right angle at <math>A</math> and <math>AD=12</math>. Points <math>C</math> and <math>D</math> are on opposite sides of <math>\overline{AB}</math>. The line through <math>D</math> parallel to <math>\overline{AC}</math> meets <math>\overline{CB}</math> extended at <math>E</math>. If
 +
<cmath>\frac{DE}{DB}=\frac{m}{n},</cmath>
 +
where <math>m</math> and <math>n</math> are relatively prime positive integers, then <math>m+n=</math>
 +
 
 +
<math>\text{(A) } 25\quad
 +
\text{(B) } 128\quad
 +
\text{(C) } 153\quad
 +
\text{(D) } 243\quad
 +
\text{(E) } 256</math>
 +
 
 +
[[1991 AHSME Problems/Problem 19|Solution]]
 +
 
 +
== Problem 20 ==
 +
 
 +
The sum of all real <math>x</math> such that <math>(2^x-4)^3+(4^x-2)^3=(4^x+2^x-6)^3</math> is
 +
 
 +
(A) 3/2  (B) 2  (C) 5/2  (D) 3  (E) 7/2
 +
 
 +
[[1991 AHSME Problems/Problem 20|Solution]]
 +
 
 +
== Problem 21 ==
 +
 
 +
 
 +
For all real numbers <math>x</math> except <math>x=0</math> and <math>x=1</math> the function <math>f(x)</math> is defined by <math>f(x/(1-x))=1/x</math>. Suppose <math>0\leq t\leq \pi/2</math>. What is the value of <math>f(\sec^2t)</math>?
 +
 
 +
<math>\text{(A) } sin^2\theta\quad
 +
\text{(B) } cos^2\theta\quad
 +
\text{(C) } tan^2\theta\quad
 +
\text{(D) } cot^2\theta\quad
 +
\text{(E) } csc^2\theta</math>
 +
 
 +
[[1991 AHSME Problems/Problem 21|Solution]]
 +
 
 +
== Problem 22 ==
 +
 
 +
<asy>
 +
draw(circle((0,6sqrt(2)),2sqrt(2)),black+linewidth(.75));
 +
draw(circle((0,3sqrt(2)),sqrt(2)),black+linewidth(.75));
 +
draw((-8/3,16sqrt(2)/3)--(-4/3,8sqrt(2)/3)--(0,0)--(4/3,8sqrt(2)/3)--(8/3,16sqrt(2)/3),dot);
 +
MP("B",(-8/3,16*sqrt(2)/3),W);MP("B'",(8/3,16*sqrt(2)/3),E);
 +
MP("A",(-4/3,8*sqrt(2)/3),W);MP("A'",(4/3,8*sqrt(2)/3),E);
 +
MP("P",(0,0),S);
 +
</asy>
 +
 
 +
 
 +
Two circles are externally tangent. Lines <math>\overline{PAB}</math> and <math>\overline{PA'B'}</math> are common tangents with <math>A</math> and <math>A'</math> on the smaller circle <math>B</math> and <math>B'</math> on the larger circle. If <math>PA=AB=4</math>, then the area of the smaller circle is
 +
 
 +
<math>\text{(A) } 1.44\pi\quad
 +
\text{(B) } 2\pi\quad
 +
\text{(C) } 2.56\pi\quad
 +
\text{(D) } \sqrt{8}\pi\quad
 +
\text{(E) } 4\pi</math>
 +
 
 +
 
 +
[[1991 AHSME Problems/Problem 22|Solution]]
 +
 
 +
== Problem 23 ==
 +
 
 +
<asy>
 +
draw((0,0)--(0,2)--(2,2)--(2,0)--cycle,dot);
 +
draw((2,2)--(0,0)--(0,1)--cycle,dot);
 +
draw((0,2)--(1,0),dot);
 +
MP("B",(0,0),SW);MP("A",(0,2),NW);MP("D",(2,2),NE);MP("C",(2,0),SE);
 +
MP("E",(0,1),W);MP("F",(1,0),S);MP("H",(2/3,2/3),E);MP("I",(2/5,6/5),N);
 +
dot((1,0));dot((0,1));dot((2/3,2/3));dot((2/5,6/5));
 +
</asy>
 +
 
 +
 
 +
If <math>ABCD</math> is a <math>2X2</math> square, <math>E</math> is the midpoint of <math>\overline{AB}</math>,<math>F</math> is the midpoint of <math>\overline{BC}</math>,<math>\overline{AF}</math> and <math>\overline{DE}</math> intersect at <math>I</math>, and <math>\overline{BD}</math> and <math>\overline{AF}</math> intersect at <math>H</math>, then the area of quadrilateral <math>BEIH</math> is
 +
 
 +
<math>\text{(A) } \frac{1}{3}\quad
 +
\text{(B) } \frac{2}{5}\quad
 +
\text{(C) } \frac{7}{15}\quad
 +
\text{(D) } \frac{8}{15}\quad
 +
\text{(E) } \frac{3}{5}</math>
 +
 
 +
[[1991 AHSME Problems/Problem 23|Solution]]
 +
 
 +
== Problem 24 ==
 +
 
 +
The graph, <math>G</math> of <math>y=\log_{10}x</math> is rotated <math>90^{\circ}</math> counter-clockwise about the origin to obtain a new graph <math>G'</math>. Which of the following is an equation for <math>G'</math>?
 +
 
 +
(A) <math>y=\log_{10}\left(\frac{x+90}{9}\right)</math> (B) <math>y=\log_{x}10</math> (C) <math>y=\frac{1}{x+1}</math> (D) <math>y=10^{-x}</math> (E) <math>y=10^x</math>
 +
 
 +
[[1991 AHSME Problems/Problem 24|Solution]]
 +
 
 +
== Problem 25 ==
 +
 
 +
If <math>T_n=1+2+3+\cdots +n</math> and
 +
<cmath>P_n=\frac{T_2}{T_2-1}\cdot\frac{T_3}{T_3-1}\cdot\frac{T_4}{T_4-1}\cdot\cdots\cdot\frac{T_n}{T_n-1}</cmath>
 +
for <math>n=2,3,4,\cdots,</math> then <math>P_{1991}</math> is closest to which of the following numbers?
 +
 
 +
<math>\text{(A) } 2.0\quad
 +
\text{(B) } 2.3\quad
 +
\text{(C) } 2.6\quad
 +
\text{(D) } 2.9\quad
 +
\text{(E) } 3.2</math>
 +
 
 +
[[1991 AHSME Problems/Problem 25|Solution]]
 +
 
 +
== Problem 26 ==
 +
 
 +
An <math>n</math>-digit positive integer is cute if its <math>n</math> digits are an arrangement of the set <math>\{1,2,...,n\}</math> and its first
 +
<math>k</math> digits form an integer that is divisible by <math>k</math>  , for  <math>k  = 1,2,...,n</math>. For example, <math>321</math> is a cute <math>3</math>-digit integer because <math>1</math> divides <math>3</math>, <math>2</math> divides <math>32</math>, and <math>3</math> divides <math>321</math>. Howmany cute <math>6</math>-digit integers are there?
 +
 
 +
<math>\text{(A) } 0\quad
 +
\text{(B) } 1\quad
 +
\text{(C) } 2\quad
 +
\text{(D) } 3\quad
 +
\text{(E) } 4</math>
 +
 
 +
 
 +
[[1991 AHSME Problems/Problem 26|Solution]]
 +
 
 +
== Problem 27 ==
 +
 
 +
If <math>x+\sqrt{x^2-1}+\frac{1}{x-\sqrt{x^2-1}}=20</math> then <math>x^2+\sqrt{x^4-1}+\frac{1}{x^2+\sqrt{x^4-1}}=</math>
 +
 
 +
(A) <math>5.05</math> (B) <math>20</math> (C) <math>51.005</math> (D) <math>61.25</math> (E) <math>400</math>
 +
 
 +
 
 +
[[1991 AHSME Problems/Problem 27|Solution]]
 +
 
 +
== Problem 28 ==
 +
 
 +
Initially an urn contains 100 white and 100 black marbles. Repeatedly 3 marbles are removed (at random) from the urn and replaced with some marbles from a pile outside the urn as follows: 3 blacks are replaced with 1 black, or 2 blacks and 1 white are replaced with a white and a black, or 1 black and 2 whites are replaced with 2 whites, or 3 whites are replaced with a black and a white. Which of the following could be the contents of the urn after repeated applications of this procedure?
 +
 
 +
(A) 2 black (B) 2 white (C) 1 black (D) 1 black and 1 white  (E) 1 white
 +
 
 +
[[1991 AHSME Problems/Problem 28|Solution]]
 +
 
 +
== Problem 29 ==
 +
 
 +
Equilateral triangle <math>ABC</math> has <math>P</math> on <math>AB</math> and <math>Q</math> on <math>AC</math>. The triangle is folded along <math>PQ</math> so that vertex <math>A</math> now rests at <math>A'</math> on side <math>BC</math>. If <math>BA'=1</math> and <math>A'C=2</math> then the length of the crease <math>PQ</math> is
 +
 
 +
(A) <math>\frac{8}{5}</math> (B) <math>\frac{7}{20}\sqrt{21}</math> (C) <math>\frac{1+\sqrt{5}}{2}</math> (D) <math>\frac{13}{8}</math> (E) <math>\sqrt{3}</math>
 +
 
 +
[[1991 AHSME Problems/Problem 29|Solution]]
 +
 
 +
== Problem 30 ==
 +
 
 +
 
 +
For any set <math>S</math>, let <math>|S|</math> denote the number of elements in <math>S</math>, and let <math>n(S)</math> be the number of subsets of <math>S</math>, including the empty set and the set <math>S</math> itself. If <math>A</math>, <math>B</math>, and <math>C</math> are sets for which <math>n(A)+n(B)+n(C)=n(A\cup B\cup C)</math> and <math>|A|=|B|=100</math>, then what is the minimum possible value of <math>|A\cap B\cap C|</math>?
 +
 
 +
(A) 96 (B) 97 (C) 98 (D) 99 (E) 100
 +
 
 +
[[1991 AHSME Problems/Problem 30|Solution]]
 +
 
 +
 
 +
== See also ==
 +
*[[AHSME]]
 +
*[[AMC12]]
 
{{MAA Notice}}
 
{{MAA Notice}}

Revision as of 16:38, 28 September 2014

Problem 1

If for any three distinct numbers $a$, $b$, and $c$ we define $f(a,b,c)=\frac{c+a}{c-b}$, then $f(1,-2,-3)$ is

$\textbf {(A) } -2 \qquad \textbf {(B) } -\frac{2}{5} \qquad \textbf {(C) } -\frac{1}{4} \qquad \textbf {(D) } \frac{2}{5} \qquad \textbf {(E) } 2$


Solution

Problem 2

$|3-\pi|=$

$\textbf{(A)\ }\frac{1}{7}\qquad\textbf{(B)\ }0.14\qquad\textbf{(C)\ }3-\pi\qquad\textbf{(D)\ }3+\pi\qquad\textbf{(E)\ }\pi-3$


Solution

Problem 3

$(4^{-1}-3^{-1})^{-1}=$

(A) $-12$ (B) $-1$ (C) $\frac{1}{12}$ (D) $1$ (E) $12$

Solution

Problem 4

Which of the following triangles cannot exist?

(A) An acute isosceles triangle (B) An isosceles right triangle (C) An obtuse right triangle (D) A scalene right triangle (E) A scalene obtuse tr iangle


Solution

Problem 5

[asy] draw((0,0)--(2,2)--(2,1)--(5,1)--(5,-1)--(2,-1)--(2,-2)--cycle,dot); MP("A",(0,0),W);MP("B",(2,2),N);MP("C",(2,1),S);MP("D",(5,1),NE);MP("E",(5,-1),SE);MP("F",(2,-1),NW);MP("G",(2,-2),S); MP("5",(2,1.5),E);MP("5",(2,-1.5),E);MP("20",(3.5,1),N);MP("20",(3.5,-1),S);MP("10",(5,0),E); [/asy]

In the arrow-shaped polygon [see figure], the angles at vertices $A,C,D,E$ and $F$ are right angles, $BC=FG=5, CD=FE=20, DE=10$, and $AB=AG$. The area of the polygon is closest to $\text{(A) } 288\quad \text{(B) } 291\quad \text{(C) } 294\quad \text{(D) } 297\quad \text{(E) } 300$

Solution

Problem 6

If $x\geq 0$, then $\sqrt{x\sqrt{x\sqrt{x}}}=$

(A) $x\sqrt{x}$ (B) $x \sqrt[4]{x}$ (C) $\sqrt[8]{x}$ (D) $\sqrt[8]{x^3}$ (E) $\sqrt[8]{x^7}$

Solution

Problem 7

If $x=\frac{a}{b}$, $a\neq b$ and $b\neq 0$, then $\frac{a+b}{a-b}=$

(A) $\frac{x}{x+1}$ (B) $\frac{x+1}{x-1}$ (C) $1$ (D) $x-\frac{1}{x}$ (E) $x+\frac{1}{x}$

Solution

Problem 8

Liquid $X$ does not mix with water. Unless obstructed, it spreads out on the surface of water to form a circular film $0.1$cm thick. A rectangular box measuring $6$cm by $3$cm by $12$cm is filled with liquid $X$. Its contents are poured onto a large body of water. What will be the radius, in centimeters, of the resulting circular film?

(A) $\frac{\sqrt{216}}{\pi}$ (B) $\sqrt{\frac{216}{\pi}}$ (C) $\sqrt{\frac{2160}{\pi}}$ (D) $\frac{216}{\pi}$ (E) $\frac{2160}{\pi}$


Solution

Problem 9

From time $t=0$ to time $t=1$ a population increased by $i\%$, and from time $t=1$ to time $t=2$ the population increased by $j\%$. Therefore, from time $t=0$ to time $t=2$ the population increased by

$\text{(A) (i+j)\%} \quad \text{(B) } ij\%\quad \text{(C) } (i+ij)\%\quad \text{(D) } \left(i+j+\frac{ij}{100}\right)\%\quad \text{(E) } \left(i+j+\frac{i+j}{100}\right)\%$

Solution

Problem 10

Point $P$ is $9$ units from the center of a circle of radius $15$. How many different chords of the circle contain $P$ and have integer lengths?

(A) 11 (B) 12 (C) 13 (D) 14 (E) 29

Solution

Problem 11

Jack and Jill run 10 km. They start at the same point, run 5 km up a hill, and reurn to the starting point by the same route. Jack has a 10 minute head start and runs at the rate of 15 km/hr uphill and 20 km/hr downhill. Jill runs 16 km/hr uphill and 22 km/hr downhill. How far from the top of the hill are they when they pass each other going in opposite directions (in km)?

$\text{(A) } \frac{5}{4}\quad \text{(B) } \frac{35}{27}\quad \text{(C) } \frac{27}{20}\quad \text{(D) } \frac{7}{3}\quad \text{(E) } \frac{28}{49}$

Solution

Problem 12

The measures (in degrees) of the interior angles of a convex hexagon form an arithmetic sequence of integers. Let $m$ be the measure of the largest interior angle of the hexagon. The largest possible value of $m$, in degrees, is

(A) 165 (B) 167 (C) 170 (D) 175 (E) 179

Solution

Problem 13

Horses $X,Y$ and $Z$ are entered in a three-horse race in which ties are not possible. The odds against $X$ winning are $3:1$ and the odds against $Y$ winning are $2:3$, what are the odds against $Z$ winning? (By "odds against $H$ winning are $p:q$" we mean the probability of $H$ winning the race is $\frac{q}{p+q}$.)

$\text{(A) } 3:20\quad \text{(B) } 5:6\quad \text{(C) } 8:5\quad \text{(D) } 17:3\quad \text{(E) } 20:3$

Solution

Problem 14

If $x$ is the cube of a positive integer and $d$ is the number of positive integers that are divisors of $x$, then $d$ could be

(A) $200$ (B) $201$ (C) $202$ (D) $203$ (E) $204$

Solution

Problem 15

A circular table has 60 chairs around it. There are $N$ people seated at this table in such a way that the next person seated must sit next to someone. What is the smallest possible value for $N$?

$\text{(A) } 15\quad \text{(B) } 20\quad \text{(C) } 30\quad \text{(D) } 40\quad \text{(E) } 58$

Solution

Problem 16

One hundred students at Century High School participated in the AHSME last year, and their mean score was 100. The number of non-seniors taking the AHSME was $50\%$ more than the number of seniors, and the mean score of the seniors was $50\%$ higher than that of the non-seniors. What was the mean score of the seniors?

(A) $100$ (B) $112.5$ (C) $120$ (D) $125$ (E) $150$

Solution

Problem 17

A positive integer $N$ is a palindrome if the integer obtained by reversing the sequence of digits of $N$ is equal to $N$. The year 1991 is the only year in the current century with the following 2 properties:

(a) It is a palindrome (b) It factors as a product of a 2-digit prime palindrome and a 3-digit prime palindrome.

How many years in the millenium between 1000 and 2000 have properties (a) and (b)?

$\text{(A) } 1\quad \text{(B) } 2\quad \text{(C) } 3\quad \text{(D) } 4\quad \text{(E) } 5$

Solution

Problem 18

If $S$ is the set of points $z$ in the complex plane such that $(3+4i)z$ is a real number, then $S$ is a

(A) right triangle (B) circle (C) hyperbola (D) line (E) parabola


Solution

Problem 19

[asy] draw((0,0)--(0,3)--(4,0)--cycle,dot); draw((4,0)--(7,0)--(7,10)--cycle,dot); draw((0,3)--(7,10),dot); MP("C",(0,0),SW);MP("A",(0,3),NW);MP("B",(4,0),S);MP("E",(7,0),SE);MP("D",(7,10),NE); [/asy]

Triangle $ABC$ has a right angle at $C, AC=3$ and $BC=4$. Triangle $ABD$ has a right angle at $A$ and $AD=12$. Points $C$ and $D$ are on opposite sides of $\overline{AB}$. The line through $D$ parallel to $\overline{AC}$ meets $\overline{CB}$ extended at $E$. If \[\frac{DE}{DB}=\frac{m}{n},\] where $m$ and $n$ are relatively prime positive integers, then $m+n=$

$\text{(A) } 25\quad \text{(B) } 128\quad \text{(C) } 153\quad \text{(D) } 243\quad \text{(E) } 256$

Solution

Problem 20

The sum of all real $x$ such that $(2^x-4)^3+(4^x-2)^3=(4^x+2^x-6)^3$ is

(A) 3/2 (B) 2 (C) 5/2 (D) 3 (E) 7/2

Solution

Problem 21

For all real numbers $x$ except $x=0$ and $x=1$ the function $f(x)$ is defined by $f(x/(1-x))=1/x$. Suppose $0\leq t\leq \pi/2$. What is the value of $f(\sec^2t)$?

$\text{(A) } sin^2\theta\quad \text{(B) } cos^2\theta\quad \text{(C) } tan^2\theta\quad \text{(D) } cot^2\theta\quad \text{(E) } csc^2\theta$

Solution

Problem 22

[asy] draw(circle((0,6sqrt(2)),2sqrt(2)),black+linewidth(.75)); draw(circle((0,3sqrt(2)),sqrt(2)),black+linewidth(.75)); draw((-8/3,16sqrt(2)/3)--(-4/3,8sqrt(2)/3)--(0,0)--(4/3,8sqrt(2)/3)--(8/3,16sqrt(2)/3),dot); MP("B",(-8/3,16*sqrt(2)/3),W);MP("B'",(8/3,16*sqrt(2)/3),E); MP("A",(-4/3,8*sqrt(2)/3),W);MP("A'",(4/3,8*sqrt(2)/3),E); MP("P",(0,0),S); [/asy]


Two circles are externally tangent. Lines $\overline{PAB}$ and $\overline{PA'B'}$ are common tangents with $A$ and $A'$ on the smaller circle $B$ and $B'$ on the larger circle. If $PA=AB=4$, then the area of the smaller circle is

$\text{(A) } 1.44\pi\quad \text{(B) } 2\pi\quad \text{(C) } 2.56\pi\quad \text{(D) } \sqrt{8}\pi\quad \text{(E) } 4\pi$


Solution

Problem 23

[asy] draw((0,0)--(0,2)--(2,2)--(2,0)--cycle,dot); draw((2,2)--(0,0)--(0,1)--cycle,dot); draw((0,2)--(1,0),dot); MP("B",(0,0),SW);MP("A",(0,2),NW);MP("D",(2,2),NE);MP("C",(2,0),SE); MP("E",(0,1),W);MP("F",(1,0),S);MP("H",(2/3,2/3),E);MP("I",(2/5,6/5),N); dot((1,0));dot((0,1));dot((2/3,2/3));dot((2/5,6/5)); [/asy]


If $ABCD$ is a $2X2$ square, $E$ is the midpoint of $\overline{AB}$,$F$ is the midpoint of $\overline{BC}$,$\overline{AF}$ and $\overline{DE}$ intersect at $I$, and $\overline{BD}$ and $\overline{AF}$ intersect at $H$, then the area of quadrilateral $BEIH$ is

$\text{(A) } \frac{1}{3}\quad \text{(B) } \frac{2}{5}\quad \text{(C) } \frac{7}{15}\quad \text{(D) } \frac{8}{15}\quad \text{(E) } \frac{3}{5}$

Solution

Problem 24

The graph, $G$ of $y=\log_{10}x$ is rotated $90^{\circ}$ counter-clockwise about the origin to obtain a new graph $G'$. Which of the following is an equation for $G'$?

(A) $y=\log_{10}\left(\frac{x+90}{9}\right)$ (B) $y=\log_{x}10$ (C) $y=\frac{1}{x+1}$ (D) $y=10^{-x}$ (E) $y=10^x$

Solution

Problem 25

If $T_n=1+2+3+\cdots +n$ and \[P_n=\frac{T_2}{T_2-1}\cdot\frac{T_3}{T_3-1}\cdot\frac{T_4}{T_4-1}\cdot\cdots\cdot\frac{T_n}{T_n-1}\] for $n=2,3,4,\cdots,$ then $P_{1991}$ is closest to which of the following numbers?

$\text{(A) } 2.0\quad \text{(B) } 2.3\quad \text{(C) } 2.6\quad \text{(D) } 2.9\quad \text{(E) } 3.2$

Solution

Problem 26

An $n$-digit positive integer is cute if its $n$ digits are an arrangement of the set $\{1,2,...,n\}$ and its first $k$ digits form an integer that is divisible by $k$ , for $k  = 1,2,...,n$. For example, $321$ is a cute $3$-digit integer because $1$ divides $3$, $2$ divides $32$, and $3$ divides $321$. Howmany cute $6$-digit integers are there?

$\text{(A) } 0\quad \text{(B) } 1\quad \text{(C) } 2\quad \text{(D) } 3\quad \text{(E) } 4$


Solution

Problem 27

If $x+\sqrt{x^2-1}+\frac{1}{x-\sqrt{x^2-1}}=20$ then $x^2+\sqrt{x^4-1}+\frac{1}{x^2+\sqrt{x^4-1}}=$

(A) $5.05$ (B) $20$ (C) $51.005$ (D) $61.25$ (E) $400$


Solution

Problem 28

Initially an urn contains 100 white and 100 black marbles. Repeatedly 3 marbles are removed (at random) from the urn and replaced with some marbles from a pile outside the urn as follows: 3 blacks are replaced with 1 black, or 2 blacks and 1 white are replaced with a white and a black, or 1 black and 2 whites are replaced with 2 whites, or 3 whites are replaced with a black and a white. Which of the following could be the contents of the urn after repeated applications of this procedure?

(A) 2 black (B) 2 white (C) 1 black (D) 1 black and 1 white (E) 1 white

Solution

Problem 29

Equilateral triangle $ABC$ has $P$ on $AB$ and $Q$ on $AC$. The triangle is folded along $PQ$ so that vertex $A$ now rests at $A'$ on side $BC$. If $BA'=1$ and $A'C=2$ then the length of the crease $PQ$ is

(A) $\frac{8}{5}$ (B) $\frac{7}{20}\sqrt{21}$ (C) $\frac{1+\sqrt{5}}{2}$ (D) $\frac{13}{8}$ (E) $\sqrt{3}$

Solution

Problem 30

For any set $S$, let $|S|$ denote the number of elements in $S$, and let $n(S)$ be the number of subsets of $S$, including the empty set and the set $S$ itself. If $A$, $B$, and $C$ are sets for which $n(A)+n(B)+n(C)=n(A\cup B\cup C)$ and $|A|=|B|=100$, then what is the minimum possible value of $|A\cap B\cap C|$?

(A) 96 (B) 97 (C) 98 (D) 99 (E) 100

Solution


See also

The problems on this page are copyrighted by the Mathematical Association of America's American Mathematics Competitions. AMC logo.png